If lot Z is the $12 lot, then on which one of the following days must Anastasia park in lot Y?

Joshua James on October 15, 2014

Could you,show me how to diagram this game?

Thanks! You guys are great.

Reply
Create a free account to read and take part in forum discussions.

Already have an account? log in

Mehran on November 7, 2014

Thanks for your message. The video setup and question explanations are now live inside of LSATMax.

Hope this helps! Please let us know if you have any other questions.